LSAT and Law School Admissions Forum

Get expert LSAT preparation and law school admissions advice from PowerScore Test Preparation.

 lawschoolforme
  • Posts: 33
  • Joined: Oct 15, 2013
|
#12467
Hello,

I agree that A is the right answer, but why is B wrong? Also, what's the difference between "in the long run" and "the foreseeable future"?

Thanks!

-lawschoolforme
 David Boyle
PowerScore Staff
  • PowerScore Staff
  • Posts: 836
  • Joined: Jun 07, 2013
|
#12475
lawschoolforme wrote:Hello,

I agree that A is the right answer, but why is B wrong? Also, what's the difference between "in the long run" and "the foreseeable future"?

Thanks!

-lawschoolforme
Hello lawschoolforme,

Is this the question starting "This company will not be training any more pilots in the foreseeable future, since we have 400 trained pilots on our waiting list who are seeking employment"? If so: it's maybe precisely because "the long run" is a lot "longer" than "the foreseeable future", maybe. "Foreseeable" may be synonymous with "not the FAR future", whereas "long run" could be a zillion years from now, so to speak.
It doesn't have to be a zillion years from now, of course. But it could. And A is just a much better answer, since if six companies need 100 pilots each, and only 400 are on the wait list…, then "Do the math!", as they say.

Hope that helps,
David
 LustingFor!L
  • Posts: 80
  • Joined: Aug 27, 2016
|
#38054
Is B wrong too, because it is attacking the first sentence, a premise, while A challenges the conclusion?
 AthenaDalton
PowerScore Staff
  • PowerScore Staff
  • Posts: 296
  • Joined: May 02, 2017
|
#38383
As David explained, answer choice (B) isn't as strong of an argument as answer choice (A). Answer choice (A) demonstrably proves that the underlying math is incorrect and new pilots will, in fact, be trained in the foreseeable future.

By contrast, answer choice (B) discusses what will happen in the "long run." This is off-point since the argument is about what will happen in the "foreseeable future." The argument in the stimulus is focused on one time period while (B) is focusing on another. It's not a strong attack.

I hope this makes sense. Good luck studying!
User avatar
 askuwheteau@protonmail.com
  • Posts: 17
  • Joined: Feb 08, 2024
|
#105809
Good morning,

I struggled with this question and I'd like some insight into why my reasoning is incorrect.

I identified the conclusion of the stimulus to be the following: "This company will not be training any more pilots in the foreseeable future".

The stimulus states that the one company has 400 ppl on the waitlist with each one of the other five companies having roughly the same number on their individual company waitlists. So, the total number of ppl on the waitlists across all six companies is 400 multiplied by 6= 2400 ppl (in total on waitlist). The stimulus further describes how projected demand for new pilots is expected to be only an additional 100 pilots for each of the five companies. Consequently, this would result in projected demand metric of 500 pilots to be needed in the industry.

Bottom line: If there are significantly more ppl on the waitilist (2400) and projected demand is only for 500 new pilots, then there appears to be a significant excess number of ppl, thus supporting the subsidiary conclusion that there will be no shortage of personnel.

So with my thought process explained above, how does answer choice A effectively weaken the conclusion: "This company will not be training any more pilots in the foreseeable future"?

Thanks in advance,

Jonathan
User avatar
 Dana D
PowerScore Staff
  • PowerScore Staff
  • Posts: 117
  • Joined: Feb 06, 2024
|
#105834
Hey there,

There are two main issues with the reasoning here. First, the conclusion is not simply that the company will not be training any more pilots - it's the entire first sentence. They will not be training any more pilots because the company has 400 trained pilots on the waitlist seeking employment.

Once you add on that additional piece of information to the conclusion, hopefully you can see how answer choice (A) would weaken that argument, because the second issue with the reasoning is the assumption that pilots cannot be on multiple waitlists.

If we add in the possibility that most of the 400 pilots on the waitlist for the company at hand are also on the waitlist for other companies, all of the sudden we go from thinking we have 2,400 people to select from to actually maybe being short on the number of pilots that we need (Each of the 6 major companies needs 100 additional pilots and there's really only about 400 trained pilots on the waitlist for the entire 6 to share). Thus, the argument for why the company doesn't need to train more people no longer makes sense.

Hope that helps!
User avatar
 askuwheteau@protonmail.com
  • Posts: 17
  • Joined: Feb 08, 2024
|
#105894
Hi Dana,

Thank you for providing such a helpful explanation. Upon reexamining the argument, I realized that the there is indeed an implicit assumption in the argument that pilots cannot be shared between waitlists (just as you said). Realizing that pilots could be shared between waitlists, thereby impugning the strength of the argument, opened my eyes to seeing the problem in its proper context.

Have a wonderful weekend,

Jonathan Sloan

Get the most out of your LSAT Prep Plus subscription.

Analyze and track your performance with our Testing and Analytics Package.